Tải bản đầy đủ (.pdf) (28 trang)

Bài giảng các bài toán về hình học tổ hợp

Bạn đang xem bản rút gọn của tài liệu. Xem và tải ngay bản đầy đủ của tài liệu tại đây (418.69 KB, 28 trang )

CÁC BÀI TỐN VỀ HÌNH HỌC TỔ HỢP
Lê Phúc Lữ - Thành phố Hồ Chí Minh
I. Kiến thức cần nhớ.
1. Các khái niệm cơ bản về hình học tổ hợp.
- Khoảng cách: từ điểm M đến hình ( H ) là min {MN | N ∈ ( H )} .
Chẳng hạn nếu ( H ) là một điểm thì khoảng cách từ M đến hình ( H ) chính là độ dài đoạn
thẳng, nếu ( H ) là đường tròn (O) thì đó chính là khoảng cách từ M đến giao điểm gần nhất
của MO với đường tròn,...

h
M

N

O

- Lân cận: bán kính d của hình ( H ) là tập hợp các điểm M có khoảng cách đến ( H ) không
vượt quá d .
Chẳng hạn: lân cận của một điểm là một hình trịn, lân cận của một đường trịn là một hình
xuyến, lân cận của một đoạn thẳng là hai hình chữ nhật và hai nửa hình trịn, lân cận của một đa
giác là gồm nhiều hình chữ nhật và nhiều phần của một hình trịn.

A

B

D

C

- Bao lồi: của một hệ điểm là đa giác lồi có đỉnh thuộc hệ điểm đã cho, có chu vi nhỏ nhất và


chứa tồn bộ hệ điểm đó. Bao lồi là một công cụ mạnh, không chỉ để giải quyết các bài tốn
mang tính lý thuyết mà cịn cả những bài mang tính thực tiễn cao.
1


- Điểm ngun: trong hệ trục tọa độ vng góc Oxy hoặc trong khơng gian Oxyz là những
điểm có tọa độ đều là các số nguyên.
2. Một số định lí cơ bản.
- Lân cận bán kính d của một đa giác có diện tích S , chu vi P có diện tích là S + pd + π d 2 .
- Một tam giác nội tiếp trong một hình chữ nhật thì có diện tích khơng vượt q

1
diện tích của
2

hình chữ nhật đó.
- Một đa giác có khoảng cách lớn nhất giữa hai điểm M , N bất kì nằm trong nó khơng vượt q
d có thể nội tiếp được trong một hình trịn có đường kính là d .

- Một đa giác có số cạnh chẵn thì tồn tại một đường chéo không song song với bất cứ cạnh nào
của đa giác.
- Định lí Pick: một đa giác lồi khơng tự cắt có a điểm ngun trên cạnh (có tính cả đỉnh) và b
a
điểm ngun nằm phía trong thì có diện tích là S = + b − 1 .
2

II. Một số bài tập áp dụng.
Bài 1: Trong mặt phẳng cho n điểm A1 , A2 , A3 ,..., An sao cho khơng có ba điểm nào thẳng
hàng và khơng có 4 điểm nào tạo thành một hình thang. Qua mỗi điểm Ai , i = 1, n , ta vẽ các
đường thẳng song song với tất cả các đoạn thẳng Aj Ak , j ≠ k , k ∈ {1, 2,3,..., n} . Tìm số tối đa

các giao điểm của các đường thẳng song song đã vẽ.
Giải.
(n − 1)(n − 2)
đường thẳng đi qua 2 trong n − 1
Xét một điểm Ai ,1 ≤ i ≤ n nào đó, có tất cả: Cn2−1 =
2
điểm cịn lại.
n(n − 1)(n − 2)
Do có tất cả n điểm nên ta có
đường thẳng trong mặt phẳng và có n − 2 đường
2
thẳng cùng song song với nhau.
Ta sẽ tìm số giao điểm tối đa của một đường thẳng d đi qua điểm Ai ,1 ≤ i ≤ n nào đó với các
đường thẳng khác cịn lại. Ngồi đường thẳng d ra,
ta cịn M
=

n(n − 1)(n − 2)
− 1 đường thẳng
2

(n − 1)(n − 2)
− 1 đường cùng đi qua Ai như đường thẳng d.
2
Do đường thẳng d song song với P= n − 3 đường thẳng khác nên số giao điểm nhiều nhất trên
đường thẳng d là:

khác và trong
đó có N
=


2


n3 − 4n 2 + 3n + 4
 n(n − 1)(n − 2)   (n − 1)(n − 2) 
.
=
− 1 − 
−=
M −N −P 
1 − (n − 3)
2
2
2

 

n(n − 1)(n − 2)
Vì có tất cả
đường thẳng và mỗi giao điểm như trên được tính 2 lần nên số giao
2
n(n − 1)(n − 2)(n3 − 4n 2 + 3n + 4)
.
8
Bài toán trên thú vị ở chỗ là nếu thay song song bởi vng góc thì vẫn cho ra cùng một kết quả
như trên.

điểm tối đa có thể có là:


Bài 2. Trong mặt phẳng cho n điểm phân biệt A1 , A2 ,..., An sao cho không có ba điểm nào
thẳng hàng và bốn điểm nào tạo thành hình bình hành. Gọi M i , i = 1, m là trung điểm của các
đoạn thẳng Ai Aj , i ≠ j nào đó. Gọi N là tổng độ dài các đoạn Ai Aj , i ≠ j và M là tổng độ dài
các đoạn M i M j , i ≠ j . Chứng minh rằng: M <

n 2 − 3n + 1
N.
2

Giải.
Trước hết, ta thấy rằng:
- Nếu M, N, P là trung điểm của các cạnh BC, CA, AB của tam giác ABC thì:
1
MN + NP + PM=
( AB + BC + CA) .
2
- Nếu M, N, P, Q, R, S là trung điểm của các cạnh AB, CD, BC, DA, AC, BD của tứ giác ABCD
thì ta có bất đẳng thức sau:
1
MN + PQ + RS < ( AB + BC + CD + DA + AC + BD)
2
B

A

M
A

S


N

P

P
R

Q

B

M

C

D

N

C

Áp dụng hai hệ thức trên cho tất cả các trung điểm. Ta thấy mỗi đoạn M i M j , i ≠ j bất kì đều
thuộc về một tam giác hoặc một tứ giác nào đó, còn mỗi cạnh Ai Aj , i ≠ j bất kì thì thuộc về
n – 2 tam giác hoặc thuộc về Cn2− 2 =

(n − 2)(n − 3)
tứ giác. Cộng từng vế các hệ thức đó, ta được:
2

3



n 2 − 3n + 2
1
1

M i M j <  (n − 2) + (n − 2)(n − 3)  ∑ Ai Aj ⇔ M <
N.
4
4
2
 1≤i < j ≤ n
1≤i < j ≤ m



4


Bài 3. Trong mặt phẳng cho 3n điểm phân biệt A1 , A2 ,... A3n sao cho khơng có ba điểm nào
thẳng hàng và khoảng cách giữa 2 điểm bất kì khơng vượt q 1. Chứng minh rằng:
1. Tồn tại đường thẳng d không đi qua bất cứ điểm nào trong các điểm A1 , A2 ,... A3n và
không song song với bất cứ đường thẳng chứa 2 điểm Ai , Aj , i ≠ j nào.
2. Giả sử khoảng cách từ các điểm A1 , A2 ,... A3n đến đường thẳng d tăng dần.
Chứng minh rằng các tam giác: A3i +1 A3i + 2 A3i +3 , i = 1, n đôi một rời nhau.
3. Chứng minh rằng tổng diện tích của n tam giác trên nhỏ hơn

1
.
2


Giải.
1. Gọi Ω là đa giác lồi chứa tất cả các điểm A1 , A2 ,... A3n . Kẻ một đường thẳng trong mặt phẳng
không cắt bất cứ cạnh nào của Ω thì đường thẳng này khơng đi qua điểm nào trong các điểm nói
trên. Do số điểm đã cho hữu hạn nên số đường thẳng qua 2 điểm bất kì cũng hữu hạn, vì vậy tồn
tại một đường thẳng khơng song song với bất cứ đường thẳng chứa 2 điểm nào trong 3n
điểmtrên.
Từ đó suy ra đường thẳng d tồn tại và ta có đpcm.
2. Qua các điểm A3i +1 , kẻ các đường thẳng ∆ i song song với d. Do khoảng cách từ các điểm
A1 , A2 ,... A3n đến d tăng dần nên các đường thẳng ∆ i nói trên chia mặt phẳng thành các dãy mà
mỗi tam giác A3i +1 A3i + 2 A3i +3 , i = 1, n được phân cách với các tam giác khác bởi các đường thẳng
∆ i . Tức là các tam giác nói trên rời nhau.

b

a

d
D
C

A
B

3. Gọi Si , i = 1, n là diện tích của tam giác A3i +1 A3i + 2 A3i +3 . Rõ ràng tồn tại 2 đường thẳng a, b cùng
vng góc với d và đi qua ít nhất 2 trong 3 đỉnh của tam giác A3i +1 A3i + 2 A3i +3 . Qua đỉnh A3i +3 , kẻ
5


một đường thẳng ∆ 'i song song với d. Kí hiệu các giao điểm như hình vẽ. Rõ ràng tam giác

A3i +1 A3i + 2 A3i +3 nằm hoàn toàn trong hình chữ nhật ABCD nên:
Si <

1
1
1
1
S ABCD =
AB.BC ≤ AC.di ≤ di ,
2
2
2
2

trong đó di là khoảng cách giữa các đường thẳng ∆ i và ∆ 'i . Suy ra:
n

n
1
1
1
S
<
di ≤ A1 A3n ≤ .


i
2
2
=i 1 =i 1 2


Ta có đpcm.
Bài 4. Trong mặt phẳng, cho tập hợp A gồm 20142 điểm phân biệt được đánh số từ 1 đến
20142 sao cho ba điểm bất kì nào trong chúng cũng không thẳng hàng. Một tứ giác (lồi hoặc
lõm) được gọi là “đẹp” nếu các đỉnh của nó thuộc A và được đánh số bằng 4 số thỏa mãn một
trong hai điều kiện sau:
-

Đó là 4 số tự nhiên cách nhau 2014 đơn vị.

-

Đó là 4 số tự nhiên liên tiếp và nếu trong đó có chứa số chia hết cho 2014 thì số đó
phải là lớn nhất hoặc nhỏ nhất.

Nối tất cả các điểm thuộc tập hợp A lại với nhau sao cho điểm nào thuộc A cũng thuộc đúng
một tứ giác.
Tìm số lớn nhất tứ giác “đẹp” được tạo thành.
Giải. Xét một bảng ơ vng gồm 2014 × 2014 ô vuông con được điền các số theo thứ tự từ trên
xuống và trái sang như sau:
1

2

3

… 2013

2014


2015 2016 2017 … 4019

4028

4029 4030 4031 … 6041

6042
























20142

Trước hết, ta sẽ chứng minh rằng không thể chia tất cả 20142 điểm đã cho thành các tứ giác
“đẹp” được. Rõ ràng 4 số trên các đỉnh của các tứ giác “đẹp” tương ứng với 4 số bị che đi trên
bảng ô vuông khi đặt một mảnh bìa hình chữ nhật kích thước 1× 4 vào đó. Ta sẽ chứng minh
rằng khơng thể che hết tồn bộ bảng ơ vng này bằng các hình chữ nhật 1× 4 .
Thật vậy, ta tơ màu các ô vuông nằm ở cột chẵn và hàng chẵn. Do bảng có 20142 ơ vng nên số
20142
ơ vng bị tô màu là:
= 1014049 là số lẻ.
4
6


Giả sử ngược lại rằng ta có thể lấp kín được cả bảng ơ vng bằng các mảnh bìa.
Khi đó, mỗi mảnh bìa sẽ che đi hoặc hai ơ vng hoặc khơng có ơ vng nào của bảng ơ vng,
tức là ln có một số chẵn ơ vng bị che đi; do đó, số ơ vng bị che đi trên bảng là một số
chẵn. Từ đó ta thấy có mâu thuẫn.
Vậy không thể che hết bảng ô vuông này bằng các hình chữ nhật 1× 4 được.
Gọi k là số tứ giác đẹp lớn nhất cần tìm thì k < 1014049 ⇒ k ≤ 1014048 .
Ta sẽ chứng minh k = 1014048 bằng cách chỉ ra cách dùng các mảnh bìa che kín bảng ơ vng.
Thật vậy, chia bảng ơ vuông thành hai phần:
- Phần 1 gồm 2012 cột đầu, ta xếp các mảnh bìa theo các hàng, mỗi hàng có đúng 503 mảnh bìa.
Khi đó, ta sẽ có thể che kín hết phần 1 bởi các mảnh bìa.
- Phần 2 gồm 2 cột cuối, ta xếp nối tiếp các mảnh bìa từ trên xuống dưới thì cuối cùng sẽ cịn lại
một ơ vng 2 × 2 ở góc dưới cùng của bảng.
Như vậy, ta dùng k = 1014048 mảnh bìa che được tối đa 20142 − 4 ơ vng của bảng.
Từ đó, ta thấy, số tứ giác “đẹp” lớn nhất cần tìm là k = 1014048 .
Bài 5. Cho một mảnh giấy hình vng. Người ta cắt mảnh giấy này thành hai mảnh hình chữ
nhật. Lấy một trong hai mảnh đó cắt làm hai mảnh sao cho đường cắt không đi qua đỉnh nào

của mảnh giấy và tiếp tục làm nhiều lần như vậy. Hỏi sau ít nhất bao nhiêu lần cắt, ta thu
được 30 đa giác có 70 cạnh?
Giải.
Giả sử sau n lần cắt, ta thu được 30 đa giác có 70 cạnh. Cần tìm giá trị nhỏ nhất của n.
Ta thấy sau một lần cắt một mảnh giấy thành 2 mảnh, số đỉnh của các đa giác tăng lên đúng 4
đỉnh (vì các đường cắt khơng đi qua đỉnh nào của mảnh giấy); do đó, sau n lần cắt, số đỉnh tăng
lên tổng cộng là 4n + 4.
Mặt khác, sau mỗi lần cắt như vậy, ta có thêm đúng 1 đa giác nên sau n lần cắt có tất cả n + 1 đa
giác; ta đã có 30 đa giác có 70 cạnh nên số đa giác còn lại là (n + 1) − 30 = n − 29 . Mà mỗi đa
giác có ít nhất 3 đỉnh (trường hợp mảnh giấy hình tam giác) nên tổng số đỉnh của các đa giác còn
lại là 3(n − 29) .
Từ đó, ta được: 4n + 4 ≥ 30.70 + 3(n − 29) ⇔ n ≥ 2009 .
Ta sẽ chứng minh n = 2009 chính là giá trị nhỏ nhất cần tìm bằng cách chỉ ra một cách cắt thỏa
mãn đề bài.
Trước hết, ta cắt các mảnh giấy 29 lần để được 30 hình chữ nhật. Sau đó, ở mỗi hình chữ nhật, ta
cắt 66 lần (mỗi lần như vậy số đỉnh của đa giác tăng lên 1) để thu được đa giác 70 cạnh. Số lần
cắt tổng cộng là: 29 + 30.66 =
2009 . Vậy số lần cắt ít nhất cần tìm là 2009 lần.
7


Bài 6. Trên mặt phẳng cho n điểm A1 , A2 , A3 ,..., An sao cho khoảng cách giữa các điểm đôi một
khác nhau. Trong các đoạn thẳng xuất phát từ Ai ,1 ≤ i ≤ n , ta gọi Aj , j ≠ i là điểm mà khoảng
cách Ai Aj ngắn nhất và tơ màu đoạn đó; tiếp tục chọn trong các đoạn thẳng xuất phát từ
Aj một đoạn Aj Ak , k ≠ i, k ≠ j ngắn nhất và tơ màu cho nó. Chứng minh rằng khi q trình

này kết thúc, khơng có đường gấp khúc khép kín nào được tơ màu tất cả các cạnh.
Giải:
Giả sử xuất phát từ điểm A1 , ta có A1 A2 là ngắn nhất, ta tô màu đoạn thẳng này. Tiếp tục xét điểm
A2 , ta có hai trường hợp xảy ra:

- Nếu min A2 Ai = A2 A1 thì q trình này dừng lại và khơng có đường gấp khúc khép kín nào được
1≤i ≤ n

tơ màu tất cả các cạnh.
- Nếu min
=
A2 Ai A2 Aj , j ≠ 1 thì ta có thể tiếp tục q trình này. Khơng mất tính tổng qt, giả sử
1≤i ≤ n

j = 3 , khi đó: A2 A3 < A1 A2 , ta tô màu đoạn A2 A3 .Tiếp tục như vậy, giả sử quá trình này dừng lại

tại điểm Ak ,3 ≤ k ≤ n , ta có các đoạn thẳng trong q trình trên đã được tơ màu là:
A1 A2 > A2 A3 > ... > Ai Ai +1 > ... > Ak −1 Ak .
Giả sử trong các đoạn này có một đường gấp khúc khép kín nào đó, tức là có một điểm nào đó là
đầu mút của hai đoạn thẳng, giả sử là điểm Ai và đường gấp khúc đó là:
Ai Ai +1... Am Ai . Khi đó, theo cách lựa chọn các điểm, ta có: Ai Ai +1 > Ai +1 Ai + 2 > ... > Am Ai . Từ đây suy
ra có điểm Am mà Am Ai < Ai Ai +1 , mâu thuẫn với cách chọn Ai +1 . Từ đây ta có đpcm.
Bài 7. Trong hình vng cạnh 200 cm có 2010 đa giác lồi mà mỗi đa giác có diện tích không
quá 2π (cm 2 ) và chu vi không quá 3π (cm) . Chứng minh trong hình vng ln tồn tại một
hình trịn có bán kính bằng 1 cm khơng cắt bất cứ đa giác nào.
Giải.
Dựng một hình vng có cạnh là 198cm bên trong hình vng kia sao cho hai hình vng này có
cùng tâm với nhau (tức là thu nhỏ mỗi cạnh của hình vng cũ 2cm). Diện tích của hình vng
cạnh 198cm này là : 1982 = 39204 (cm 2 ) .
Dựng các lân cận bán kính 1 cm của các hình đa giác đã cho. Diện tích các lân cận đó chính bằng
tổng của diện tích đa giác, diện tích các hình chữ nhật và diện tích các hình quạt dựng tại các
đỉnh của đa giác. Ta sẽ chứng minh rằng với một đa giác n cạnh bất kì, tổng các góc ngồi bằng
3600 . Thật vậy, với mỗi đa giác như vậy, ta chia thành n – 2 tam giác có chung đỉnh và đơi một
rời nhau. Tổng các góc của mỗi tam giác như vậy là 1800 nên tổng các góc trong của đa giác lồi
đó là (n − 2)1800 . Do đó, tổng các góc ngồi của đa giác lồi là : n.1800 − (n − 2).1800 =

3600 .
8


Tổng các góc của các hình quạt là 3600 , hơn nữa các hình quạt này có cùng bán kính 1 cm nên
tổng diện tích các hình quạt này chính bằng diện tích của hình trịn bán kính là 1 cm và là :

π .12 = π (cm 2 ) . Do đó, tổng diện tích mỗi đa giác và lân cận là : 2π + 3π .1 + π =
6π (cm 2 ) ;
suy ra : tổng diện tích các đa giác và lân cận là:
2010.6π < 2010.6.3, 2 = 38592 (cm 2 ) < 39204 (cm 2 ) .

Vậy tồn tại một điểm A thuộc hình vng cạnh 198 cm mà không thuộc bất cứ đa giác và lân cận
1 cm nào của chúng. Khi đó đường trịn (A,1 cm) không cắt bất cứ đa giác nào và rõ ràng nó thỏa
mãn đề bài. Ta có đpcm.
Bài 8. Trong một hình vng có cạnh là 1 cho k điểm phân biệt bất kì (có thể thẳng hàng).
Nối các điểm đó lại với nhau bởi các đoạn thẳng sao cho hai đoạn bất kì khơng cắt nhau tạo
thành các tam giác chia hình vng thành các phần nhỏ hình tam giác rời nhau. Tìm số k
nhỏ nhất sao cho trong các tam giác được chia ra, luôn tồn tại một tam giác có diện tích
1
khơng q
.
100
Giải.
9


Giả sử với k điểm nằm trong hình vng đó, có tất cả a tam giác được tạo thành. Tổng tất cả các
góc trong của các tam giác đó là a.1800 . Mặt khác, các góc đó cũng chính là các góc quay xung
quanh mỗi điểm đã cho cùng với các đỉnh của hình vng nên tổng trên cũng bằng :

4.900 + k .3600 .

Do đó, ta có 4.900 + k .3600 = a.1800 ⇔ a = 2k + 2 . Tức là với mọi cách sắp xếp các điểm và nối
các đoạn thẳng ứng với k điểm ta luôn thu được 2k + 2 tam giác.
Vì có tất cả 2k + 2 tam giác nên diện tích của tam giác nhỏ nhất không vượt quá


1
.
2k + 2

1
1

⇔ k ≥ 49 nên tất cả các giá trị k ≥ 49 đều thỏa mãn đề bài.
2k + 2 100

Ta sẽ chứng minh rằng k = 48 (hoặc nhỏ hơn) không thỏa mãn đề bài bằng cách chỉ ra một cách
sắp xếp các điểm này phía trong hình vng và cách nối chúng lại thành các tam giác mà diện
1
tích của tam giác bất kì đều lớn hơn
.
100

Thật vậy, ta đặt 48 điểm này chia đều một đường chéo đi qua hai đỉnh nào đó của hình vng và
nối các điểm này với hai đỉnh cịn lại của hình vng. Rõ ràng hình vng đã được chia thành 98
1
1
tam giác nhỏ bằng nhau và diện tích mỗi tam giác này cùng bằng
, không thỏa mãn đề

>
98 100
bài. Vậy giá trị nhỏ nhất của k cần tìm là 49.

10


Bài 9. Trong không gian cho 7 điểm phân biệt sao cho khơng có bốn điểm nào đồng phẳng.
Tất cả các điểm đó được nối với nhau bởi các đoạn thẳng. Mỗi đoạn thẳng được tô bởi hai
màu xanh, đỏ hoặc khơng được tơ màu. Tìm số k nhỏ nhất sao cho với mọi cách tô màu k
đoạn thẳng bất kì trong các đoạn thẳng đó, ln tồn tại một tam giác có ba cạnh cùng màu.
Giải.
Trước hết, ta có kết quả quen thuộc sau : Cho 6 điểm phân biệt sao cho khơng có ba điểm nào
thẳng hàng. Nếu ta tô màu tất cả các đoạn thẳng nối các điểm này bởi hai màu xanh hoặc đỏ thì
ln tồn tại một tam giác có các cạnh cùng màu.
Từ giả thiết khơng có bốn điểm nào đồng phẳng, ta suy ra rằng khơng có ba điểm nào thẳng hàng
và hai đoạn thẳng bất kì chỉ cắt nhau tại đầu mút chung (nếu có) của chúng.
Gọi 7 điểm đã cho là A1 , A2 ,..., A7 . Ta thấy với 7 điểm này, có tất cả C72 = 21 đoạn thẳng.
Do đó : k ≤ 21 . Nếu tơ màu tất cả 21 đoạn thẳng này thì chỉ cần chọn ra 6 điểm trong đó cũng sẽ
thỏa mãn điều kiện theo kết quả ở trên. Ta thử tìm giá trị k nhỏ hơn.
- Với k = 20, ta tô màu 20 đoạn và không tô màu 1 cạnh, giả sử là A1 A7 , khi đó tất cả các đoạn
thẳng trong bộ 6 điểm A1 , A2 , A3 , A4 , A5 , A6 (hoặc A2 , A3 , A4 , A5 , A6 , A7 ) đều được tô màu, lại theo
kết quả trên, điều kiện được thỏa mãn, tức là k = 20 vẫn thỏa mãn đề bài.
A3

A

A

2


4

A

5

A

1

A

7

A

6

- Với k = 19, ta tô màu 19 đoạn và không tô màu 2 cạnh. Ta sẽ chỉ ra một cách tơ màu bỏ đi hai
đoạn thẳng và khơng có hai tam giác nào được tô cùng màu như trên hình vẽ.
Nếu hai cạnh A1 A7 và A2 A6 khơng được tơ màu thì trong các đoạn xuất phát từ A3, tô xanh bốn
đoạn và tô đỏ 2 đoạn ; với các đỉnh cịn lại tơ xanh 3 đoạn, tơ đỏ 3 đoạn (hoặc 2 đoạn đối với hai
điểm A6, A7).
Do đó, k = 19 khơng thỏa mãn đề bài. Rõ ràng nếu k = 19 khơng thỏa thì tất cả số nhỏ hơn nó
cũng khơng thỏa mãn.
Vậy giá trị nhỏ nhất của k cần tìm là 20.
11



Bài 10. Một đa giác đều 9 cạnh có các đỉnh được tô bởi một trong hai màu : xanh hoặc đỏ.
Chứng minh rằng tồn tại hai tam giác có đỉnh là đỉnh của đa giác đã cho và thỏa mãn hai
điều kiện sau:
i) Diện tích bằng nhau.
ii) Tất cả các đỉnh của hai tam giác đều được tô cùng màu.
Lời giải.
Ta thấy rằng tô màu 9 đỉnh của đa giác bởi 2 màu thì theo ngun lí Dirichlet, tồn tại ít nhất 5
đỉnh được tơ cùng màu. Ta sẽ chứng minh rằng trong 5 đỉnh đó, tồn tại ít nhất 2 tam giác có cùng
diện tích.
Số tam giác tạo thành từ 5 đỉnh đó là số cách chọn 3 đỉnh bất kì trong 5 đỉnh và có C53  10 tam
giác. Ta xét các phép quay đa giác quanh tâm đường tròn ngoại tiếp O của đa giác với các góc có
k 2
số đo là
, k  1, 2,3,...,9 .
9

Rõ ràng các phép quay này tuy thay đổi vị trí các đỉnh nhưng các tam giác vẫn có màu như cũ
nên sau 9 phép quay, ta thu được 90 tam giác được tô cùng màu.
Tuy nhiên, số tam giác tạo thành bởi các đỉnh của đa giác đã cho là C93  84  90 nên theo
nguyên lí Dirichlet thì tồn tại hai tam giác được tơ cùng màu là ảnh của nhau qua một phép quay
trong các phép quay đã xét, tức là chúng có diện tích bằng nhau.
Từ đó, ta có đpcm.
Bài 11. (VN TST 1999) Cho một đa giác lồi (H). Chứng minh rằng với mỗi số thực a ∈ (0,1) ,
tồn tại 6 điểm phân biệt nằm trên các cạnh của (H), kí hiệu là A1 , A2 ,..., A6 theo chiều kim
đồng hồ thỏa mãn đồng thời hai điều kiện sau:
 

1. A1 A2= A5 A4= a ⋅ A6 A3 .
2. Các đường thẳng A1 A2 , A5 A4 cách đều đường thẳng A6 A3 .
12



Giải.
Trước hết, ta sẽ dựng đoạn A6 A3 có các đỉnh nằm trên các cạnh của (H). Lấy một điểm X bất kì
nằm trong đa giác (H) đã cho, gọi (d) là một đường thẳng bất kì qua X, khơng song song với các
cạnh của (H) và A6 , A3 là hai giao điểm của (d) với các cạnh của (H). Đánh dấu một miền mặt
phẳng nằm trong (H) được chia ra bởi (d) là P và phần còn lại là Q. Ta sẽ tìm được vơ số bộ sáu
điểm thỏa mãn (1) và cần chứng minh rằng trong số đó, tồn tại ít nhất một bộ thỏa mãn (2).

A3

X
I

A6

N

M
A2

A1


Trên đoạn A6 A3 đã dựng, lấy điểm I sao cho A6 I= a ⋅ A6 A3 . Trong miền P, dựng hình bình hành
A6 IMN bất kì sao cho một trong hai đỉnh M và N nằm trên cạnh của (H). Tiếp tục tịnh tiến

vector MN về phía cạnh của (H) đến khi cả hai đỉnh đều nằm trên các cạnh của (H). Dễ thấy
đoạn MN ln tồn tại vì:
-Nếu (d) không song song với cạnh nào của (H) nên khi a → 0 thì đoạn MN cũng tiến gần về

một đỉnh nào đó của (H).
- Nếu (d) song song với một cạnh của (H), và độ dài của MN lớn hơn độ dài cạnh đó thì M, N lần
lượt thuộc về hai cạnh kề với cạnh của (H) nêu trên; ngược lại thì MN sẽ nằm ngay trên cạnh đó.
Tiếp theo, ta sẽ chứng minh rằng trong trường hợp (d) khơng song song với cạnh nào của (H) thì
trong miền (P), chỉ có đúng một đoạn MN thỏa mãn cách dựng trên.
Thật vậy, giả sử tồn tại hai đoạn MN , M ' N ' có đỉnh đều nằm trên các cạnh của (H) và các tứ
giác A6 IMN , A6 IM ' N ' là hình bình hành. Ta sẽ chứng minh rằng điều này mâu thuẫn với giả
thiết (H) là đa giác lồi.
Thật vậy, ta xét ba trường hợp:
(1) Nếu giao điểm của hai đường thẳng đi qua hai cạnh chứa A6 , A3 cắt nhau tại một điểm nằm
trong miền (P). Khi đó, các đoạn thẳng song song với A6 A3 có đỉnh nằm trong miền này đều có
độ dài nhỏ hơn A6 A3 . Nếu tồn tại hai đoạn thẳng MN , M ' N ' như trên.
13


Ta lại tiếp tục có ba trường hợp:
(1.1) Nếu có một cặp đỉnh nào đó trong chúng thuộc cùng một cạnh, giả sử là M và M’ và hai
đỉnh còn lại là N, N’ thì khơng; khi đó rõ ràng các đỉnh nằm giữa N, N’ sẽ không nằm trong cùng
nửa mặt phẳng bờ là đường thẳng đi qua đầu mút của đoạn thẳng chứa hai đỉnh N, N’, điều này
mâu thuẫn với giả thiết (H) là đa giác lồi.
(1.2) Nếu cả hai cặp đỉnh của hai đoạn MN , M ' N ' cùng nằm trên các cạnh của (H) thì hai cạnh
đó phải song song với nhau, ta thấy điều này cũng mâu thuẫn.
(1.3) Nếu khơng có cặp đỉnh nào cùng thuộc một đoạn thì những cũng tương tự trường hợp trên,
các đỉnh nằm giữa hai đỉnh M, M’ và N, N’ sẽ không thuộc cùng một nửa mặt phẳng có bờ là
đường thẳng đi qua đầu mút của đoạn thẳng chứa hai đỉnh M, M’ và N, N’; mâu thuẫn.
(2) Nếu giao điểm của đường thẳng đi qua hai cạnh chứa A6 , A3 song song thì khi tịnh tiến vector


A6 A3 trên đường thẳng đó, ta ln nhận được các vector có bằng với A6 A3 . Do đa giác đã cho là
lồi nên các đỉnh còn lại thuộc miền (P) của (H) sẽ chỉ nằm trong phần giữa hai đường thẳng song

song nêu trên. Từ đó, ta cũng đưa về trường hợp đầu tiên và cũng dẫn đến điều mâu thuẫn.

A1
A2
A6
I
A5

A3
A4

(3) Nếu giao điểm của hai đường thẳng đi qua hai cạnh chứa A6 , A3 cắt nhau tại một điểm không
nằm trong miền (P). Khi đó, độ dài của các đoạn thẳng song song với A6 A3 sẽ tăng lên đến một
giá trị nào đó rồi giảm xuống đến 0; khi nó giảm đến độ dài đúng bằng A6 A3 thì ta lại quay về
trường hợp đầu tiên và cũng có điều vơ lí.
Tiếp theo, ta cần chứng minh rằng tồn tại một cách dựng đường thẳng (d) mà các cả 6 điểm này
đều thỏa mãn đề bài.
- Nếu như khoảng cách từ A1 A2 và từ A5 A4 đến A6 A3 bằng nhau thì bài tốn kết thúc.

14


-Nếu như khoảng cách từ A1 A2 và từ A5 A4 đến A6 A3 không bằng nhau và giả sử đoạn A1 A2 gần
A6 A3 hơn đoạn A5 A4 .
Quay (d) quanh X nửa vòng tròn, độ dài của A6 A3 thay đổi liên tục như khi quay (d) quanh X cả
một vòng tròn. Độ dài của các đoạn A1 A2 và A5 A4 cũng thay đổi theo nhưng tỉ lệ giữa chúng thì
vẫn giữ nguyên. Đến khi quay xong thì các đoạn A1 A2 và A5 A4 đã đổi chỗ cho nhau, tức là
khoảng cách từ đoạn đoạn A5 A4 đến A6 A3 gần hơn đoạn A1 A2 .
Do phép biến đổi này thực hiện liên tục nên tồn tại một thời điểm mà khoảng cách giữa A1 A2 và
A5 A4 đến A6 A3 bằng nhau. Ngay lúc đó, bộ 6 điểm A1 , A2 ,..., A6 này thỏa mãn tất cả các điều

kiện của đề bài. Ta có đpcm.
Bài 12. (VN TST 2007) Cho đa giác 9 cạnh đều (H). Xét ba tam giác với các đỉnh là các đỉnh
của đa giác (H) đã cho sao cho không có hai tam giác nào có chung đỉnh. Chứng minh rằng
có thể chọn được từ mỗi tam giác 1 cạnh sao cho 3 cạnh này bằng nhau.
Lời giải.
Kí hiệu hình (H) đã cho là đa giác A1 A2 A3 ... A8 A9 như hình vẽ. Trước hết, ta thấy rằng độ dài các
cạnh và các đường chéo của hình (H) chỉ thuộc 4 giá trị khác nhau (nếu gọi R là bán kính đường
trịn ngoại tiếp của (H) thì ta dễ dàng tính được các giá trị đó là
π



, 2 R sin
, 2 R sin
) ta đặt chúng là a1 , a2 , a3 , a4 theo thứ tự tăng dần của
2 R.sin , 2 R sin
9
9
9
9
độ dài. Rõ ràng các tam giác có đỉnh thuộc các đỉnh của (H) sẽ có cạnh có độ dài thuộc 1 trong 6
dạng sau: (a1 , a1 , a2 ), (a2 , a2 , a4 ), (a1 , a3 , a4 ), (a3 , a3 , a3 ), (a2 , a3 , a4 ), (a4 , a4 , a1 ) .

A1

a1

A2
a2


A

9

A3

a3
a4

A

A4

8

A

A

7

5

A

6

15



Giả sử 3 tam giác được lấy ra là ∆1 , ∆ 2 , ∆ 3 .
Xét các trường hợp sau:
- Nếu trong các tam giác đó có một tam giác đều, rõ ràng, tam giác này phải có độ dài các cạnh

là 2 R.sin
; khơng mất tính tổng qt, giả sử đó là tam giác A1 A4 A7 . Do các tam giác
9
∆1 , ∆ 2 , ∆ 3 khơng có hai đỉnh nào trùng nhau nên ta sẽ lập một tam giác có các đỉnh là một trong
hai đỉnh của các tập hợp { A2 , A3 },{ A4 , A5 },{ A7 , A8 } . Ta sẽ chứng minh rằng tam giác đó phải có ít

, tức là hai đỉnh có chỉ số có cùng số dư khi chia cho 3. Giả sử
9

, khi đó đỉnh
ngược lại, trong hai tam giác cần lập, khơng có tam giác nào có cạnh là 2 R.sin
9
A2 phải nối với A4 và A4 phải nối với A8, nhưng khi đó A8 được nối với A2 là hai đỉnh có chỉ số
chia cho 3 cùng dư là 2, mâu thuẫn. Do đó, trong hai tam giác lập được, ln có một cạnh có độ

dài là 2 R.sin
. Suy ra trường hợp này ln có tam giác thỏa mãn đề bài.
9

nhất 1 cạnh có độ dài là 2 R.sin

- Nếu trong các tam giác đó, khơng có tam giác nào đều . Khi đó các tam giác được xét khơng có
ba đỉnh cùng thuộc một trong ba tập hợp sau: α1 = { A1 , A4 , A7 } , α 2 = { A2 , A5 , A8 } ,

α 3 = { A3 , A6 , A9 } . Ta thấy một đoạn thẳng nối hai điểm bất kì thuộc hai tập khác nhau sẽ nhận 1
trong 3 giá trị là a1 , a2 , a4 . Hơn nữa, không có tam giác nào có độ dài 3 cạnh là (a1 , a2 , a4 ) nên ta

có hai nhận xét:
(1) Một tam giác có các đỉnh thuộc cả ba tập α1 , α 2 , α 3 nói trên thì sẽ có hai cạnh nào đó có độ
dài bằng nhau (các cạnh của nó có thể là (a1 , a1 , a2 ) , (a2 , a2 , a4 ) , (a4 , a4 , a1 ) ) tức là nó phải cân.
(2) Một tam giác có hai trong ba đỉnh thuộc cùng một tập thì tam giác đó các cạnh có độ dài
là (a2 , a3 , a4 ) hoặc là (a1 , a3 , a4 ) , tức là tam giác đó khơng cân.
* Ta xét tiếp các trường hợp (các tam giác xét dưới đây là cân nhưng khơng đều):
+ Có hai tam giác cân và một tam giác khơng cân: khi đó theo nhận xét (1), hai tam giác cân đó
phải có đỉnh thuộc các tập hợp khác nhau trong ba tập α1 , α 2 , α 3 ; khi đó, rõ ràng tam giác cịn lại
cũng phải có đỉnh thuộc các tập hợp khác nhau, tức là nó phải cân, mâu thuẫn.
Vậy trường hợp này khơng tồn tại.
+ Có một tam giác cân và hai tam giác khơng cân: khi đó theo nhận xét (2), hai tam giác khơng
cân đó phải có hai đỉnh thuộc cùng một tập hợp và đỉnh còn lại thuộc tập hợp khác, giả sử một
tam giác có hai đỉnh thuộc α1 và một đỉnh thuộc α 2 ; rõ ràng tam giác khơng cân cịn lại phải có
16


hai đỉnh thuộc α 2 , một đỉnh thuộc α 3 , suy ra tam giác cịn lại có hai đỉnh thuộc α 3 , một đỉnh
thuộc α1 nên nó là tam giác cân, mâu thuẫn.
Vậy tương tự như trên, trường hợp này không tồn tại.
+ Cả ba tam giác đều khơng cân: khi đó theo nhận xét (2), tam giác đó thuộc một trong hai dạng
(a2 , a3 , a4 ) hoặc là (a1 , a3 , a4 ) , tức là các tam giác này luôn chứa 1 cạnh có độ dài là a3.
Trong trường hợp này, bài toán được giải quyết.
+ Cả ba tam giác đều cân: khi đó, các tam giác có độ dài là (a1 , a1 , a2 ) , (a2 , a2 , a4 ) , (a4 , a4 , a1 ) .
Rõ ràng khơng tồn tại trường hợp có độ dài các cạnh lần lượt nhận cả ba giá trị như ba bộ trên
nên phải có hai bộ trùng nhau, tức là có ít nhất hai tam giác cân bằng nhau và một tam giác cân
nhận một trong ba giá trị thuộc một trong các bộ trên làm cạnh, khi đó ln có thể chọn từ tam
giác này một cạnh bằng với cạnh đáy hoặc cạnh bên của hai tam giác cân bằng nhau kia.
Trong trường hợp này, bài toán cũng được giải quyết.
Vậy trong mọi trường hợp, ta ln có đpcm.
Bài 13. (Đề thi Nữ sinh châu Âu 2012) Một hình vuông đơn vị được chia thành các đa giác,

sao cho mỗi cạnh của đa giác đều song song với cạnh của hình vng cho trước. Nếu tổng độ
dài các đoạn thẳng nằm bên trong hình vng (khơng tính hình vng) là 2n (với n là một
1
số thực dương), chứng minh rằng tồn tại một đa giác có diện tích lớn hơn
.
2
( n + 1)
Giải.
Trước hết ta xét bổ đề sau : Một đa giác R được tạo thành như đề bài có diện tích a thì nó có
chu vi ít nhất là 4 a .
Chứng minh: Xét bao lồi hình chữ nhật nhỏ nhất chứa hình trên. Gọi hai kích thước bao lồi là
x và y . Do bao lồi đang xét là hình chữ nhật nhỏ nhất nên biên của R đều nằm trên các cạnh
hình chữ nhật.
Dễ thấy, do đa giác được tạo thành từ các đường thẳng song song với cạnh hình vng đơn vị
nên chu vi đa giác cũng bằng chu vi hình vng (vì các khoảng trống giữa R và bao lồi là các
hình chữ nhật).
Áp dụng bất đẳng thức AM-GM suy ra x + y ≥ 2 =
xy 2 S hcn ≥ 2 a
Do đó: PR = Phcn = 2 ( x + y ) ≥ 4 a . Bổ đề được chứng minh xong.
Đặt diện tích và chu vi tương ứng của các hình là a1 , a2 ,..., ak và p1 , p2 ,..., pk (lưu ý, các hình ta
xét khơng có diện tích chung).
17


Giả sử ngược lại tất cả các hình đều có diện tich nhỏ hơn
i = 1, 2,..., k . Mặt khác ta có:=
S hv
k

1


( n + 1)

2

, dễ thấy ai ≤

ai
với mọi
( n + 1)

k

ai 1 kết hợp với bổ đề vừa chứng minh thì thu được
∑=
i =1

k

ai

k

1

pi

n

∑ a < ∑ ( n + 1) ≤ 4 ∑ ( n + 1) =

( n + 1)

i
=i 1 =i 1

=i 1

(do tổng độ dài các đoạn thẳng là 2n , mỗi đoạn thẳng được tính hai lần vào hai chu vi của hai
hình kề nhau).
n
(đây là điều vơ lí) , dẫn đến điều chúng ta giả sử là sai, và phải tồn tại một hình
n +1
1
.
có diện tích lớn hơn
2
( n + 1)

Do đó 1 <

Ta có đpcm.
Bài 14. (VN TST 2006) Trong khơng gian cho 2006 điểm mà trong đó khơng có 4 điểm nào
đồng phẳng. Người ta nối tất cả các điểm đó lại bởi các đoạn thẳng. Số nguyên dương m gọi
là số tốt nếu ta có thể gán cho mỗi đoạn thẳng trong các đoạn thẳng đã nối bởi một số tự
nhiên không vượt quá m sao cho mỗi tam giác tạo bởi ba điểm bất kì trong số các điểm đó đều
có hai cạnh được gán bởi hai số bằng nhau và cạnh còn lại gán bởi số lớn hơn hai số đó.
Tìm số tốt có giá trị nhỏ nhất.
Lời giải.
Do trong các điểm đã cho khơng có bốn điểm nào đồng phẳng nên ba điểm bất kì trong chung
ln tạo thành một tam giác. Gọi S(n) là giá trị nhỏ nhất của số tốt ứng với n điểm trong không

gian (n là số tự nhiên), ta sẽ xác định giá trị của S(2006). Ta chỉ xét các giá trị n ≥ 4 .
1

2
2

1

1

1

-

Với n = 4 thì thử trực tiếp, ta thấy S(4) = 2. Bởi vì S(4) = 1 khơng thỏa mãn nên S (4) ≥ 2 , ta
sẽ chỉ ra rằng S(4) = 2 thỏa mãn. Cụ thể ta có thể gán các đoạn thẳng như sau : gán 4 đoạn
bất kì bởi số 1 và 2 đoạn còn lại bởi số 2, rõ ràng các tam giác tạo thành đều thỏa mãn đề bài.
18


-

Với một giá trị n > 4 bất kì, ta sẽ chứng minh rằng :
  n + 1 
S ( n) ≥ 1 + S  
 .
 2 

Gọi a là số nhỏ nhất được gán cho các đoạn thẳng trong trường hợp có n điểm. Trong trường hợp
tối thiểu, khơng mất tính tổng qt, ta giả sử rằng a = 1, ta gọi hai đầu mút của đoạn thẳng nào

đó được gán số 1 là X và Y.
Trong n – 2 điểm cịn lại, nếu có một điểm được nối với X và Y bởi một đoạn thẳng gán bởi số 1
thì điểm đó cùng với X và Y sẽ tạo thành một tam giác đều không thỏa mãn đề bài.
Do đó, nếu gọi A là tập hợp tất cả các điểm nối với X bởi một đoạn thẳng gán số 1 (có tính ln
điểm Y) và B là tập hợp tất cả các điểm nối với Y bởi một đoạn thẳng gán số 1 (có tính ln
điểm X) thì giữa A và B khơng có phần tử nào chung hay A + B =
n.
Ta có các nhận xét sau :
-

Nếu lấy một điểm bất kì trong tập A và một điểm bất kì trong B thì hai điểm đó cũng phải
được nối bởi đoạn thẳng gán số 1 vì nếu khơng thì hai điểm đó sẽ cùng với X sẽ tạo thành
một tam giác không thỏa mãn đề bài (tam giác đó hoặc khơng có hai số được gán trên hai
cạnh bằng nhau hoặc có hai cạnh bằng nhau nhưng cạnh còn lại gán số 1 nhỏ hơn).

-

Hai điểm bất kì trong A được nối với nhau bởi một đoạn thẳng gán số lớn hơn 1 bởi nếu
khơng thì khi chọn thêm một điểm trong B, ta sẽ có một tam giác khơng thỏa mãn đề bài
(tam giác đó đều). Tương tự với tập hợp B. Tức là trong các tập A và B đều có chứa các
số lớn hơn 1.

Tiếp theo, ta lại thấy trong mỗi tập A, B như vậy đều cần thêm S ( A ), S ( B ) số nữa để gán cho
 n − 1
 n + 1
.
các đoạn thẳng. Giả sử A ≥ B thì A ≥ 
+ 1 =

 2 

 2 

Ta hồn tồn có thể gán các số ở tập A trùng với các số ở tập B nên các số cần có thêm nữa là
  n + 1 
S
  , tính thêm số 1 nhỏ nhất đã được gán cho đoạn XY ban đầu, ta được:
 2 
  n + 1 
S ( n) ≥ 1 + S  
  với mọi n.
 2 

Từ đó, áp dụng liên tiếp kết quả này, ta có: (chú ý rằng S(4) = 2).
S (2006) ≥ 1 + S (1003) ≥ 2 + S (502) ≥ ... ≥ 9 + A(4) =
11 .

Tiếp theo, ta sẽ chứng minh rằng giá trị 11 này thỏa mãn đề bài. Thật vậy :
Ta xây dựng cách gán các điểm từ thấp đến cao bằng cách ghép các bộ điểm ít hơn lại. Cụ thể
như sau :
19


-

Đầu tiên ta xây dựng cho bộ 4 điểm. Cách gán tương tự như ở trên, nhưng trong trường hợp
này gán 4 đoạn bởi số 11 và 2 đoạn bởi số 10.

-

Ghép 2 bộ này lại và tách ra từ một trong hai bộ đó ra 2 điểm, gán cho đoạn thẳng nối 2 điểm

đó bởi số 10, ta đã có tất cả 8 điểm.

-

Tiếp tục ghép tương tự như vậy theo thứ tự như sau :

4 → 8 → 16 → 32 → 63 → 126 → 251 → 502 → 1003 → 2006
(Các trường hợp từ 32 đến 63 hoặc tương tự ta phải bỏ đi 1 điểm nào đó ở một trong hai bộ ra
ngồi). Mỗi lần ghép hai bộ điểm lại thì số gán trên đoạn được tách ra lại giảm đi 1 đơn vị, đến
khi ghép được 2006 điểm thì số đó chính là 1.
Dễ thấy cách gán số cho các đoạn thẳng này thỏa mãn đề bài.
Vậy giá trị nhỏ nhất của số tốt cần tìm là 11.
Bài 15. (VN TST 2000) Xét 2000 đường trịn bán kính r = 1 trên mặt phẳng sao cho khơng có
hai đường trịn nào tiếp xúc với nhau và mỗi đường trịn cắt ít nhất với hai đường trịn khác.
Hãy tìm giá trị nhỏ nhất của số giao điểm của các đường tròn này.
Giải.
Gọi G là tập hợp các đường tròn trong mặt phẳng thỏa mãn điều kiện đã cho và S là tập hợp tất
cả các giao điểm của chúng.
Với mọi C ∈ G, x ∈ S , ta xét hàm số f ( x, C ) , trong đó:


f ( x, C ) = 0 nếu x ∉ C .



f ( x, C ) =

1
nếu x ∈ C và có đúng k đường trịn đi qua x.
k


Với mỗi x ∈ S , ta thấy rằng

∑ f ( x, C ) = 1 .

C∈G

Với mỗi C ∈ G , chọn một điểm x ∈ C ∩ S sao cho f ( x, C ) =

1
đạt giá trị nhỏ nhất.
k

Gọi C1 , C2 , C3 ,..., Ck −1 là các đường tròn khác C cùng đi qua x. Do mỗi đường trịn cắt ít nhất hai
đường trịn khác nên
với i 1, 2,3,..., k − 1 thì gọi x1 , x2 , x3 ,..., xk −1 tương ứng là các giao điểm
=
khác x của các đường trịn nói trên với C.
Do các đường trịn có bán kính khác nhau nên xi ≠ x j với mọi i ≠ j ,1 ≤ i, j ≤ k − 1 .
Suy ra:

1

1

1.
∑ f ( x, C ) ≥ k + (k − 1) k =
x∈S

Từ đó, ta thấy rằng N =S =

∑ ∑ f ( x, C ) =∑ ∑ f ( x, C ) ≥ G .1 =2000 .
x∈S C∈G

C∈G x∈S

20


Gọi r là bán kính của các đường trịn. Dễ thấy rằng bốn đường trịn có bán kính r và tạo ra đúng 4
giao điểm nếu tâm của ba đường trịn tạo thành một tam giác đều có cạnh là r 3 và tâm còn lại
là trọng tâm của tam giác đều ấy.
Do đó, ta chia 2000 đường trịn thành 500 nhóm, mỗi nhóm có 4 đường trịn với 4 giao điểm và
mỗi nhóm như thế khơng có điểm chung với nhau.
Vậy số giao điểm nhỏ nhất cần tìm chính là 2000.
Bài 16. (Đề trường Xuân 2013) Các đường chéo của một ngũ giác lồi đôi một cắt nhau chia ngũ
giác thành một ngũ giác nhỏ và 10 tam giác con. Xác định số lớn nhất các tam giác con trong 10
tam giác đó mà có diện tích đơi một bằng nhau.
Giải.
Xét một ngũ giác ABCDE tương ứng với các tam giác con được chia ra như hình trên được
đánh số từ 1 đến 10. Ta sẽ chứng minh rằng trong số này có khơng q 7 tam giác con có diện
tích đơi một bằng nhau.

A

10
E

D

N


Q
P
4

B
2

R M

5
9

6

1

7

3

8
C

Dễ thấy rằng các bộ (1, 2, 6), (2,3, 7), (3, 4,8), (4,5,9), (5,1,10) khơng thể có diện tích đơi một bằng
nhau vì chẳng hạn nếu các tam giác 1, 2, 6 có diện tích đơi một bằng nhau thì suy ra tứ giác
ABNR là hình bình hành và AR  BN , mâu thuẫn. Các trường hợp cịn lại tương tự.

Do đó, có nhiều nhất 7 tam giác con có diện tích đơi một bằng nhau, đặt tập hợp các tam giác
này là A và 3 tam giác con có diện tích khác các tam giác này, đặt tập hợp các tam giác này là

B . Rõ ràng các tam giác thuộc tập hợp B phải có mặt trong cả 5 bộ trên.
Ta xét hai trường hợp của tập hợp B :

21


(1) Trường hợp B ⊂ {1, 2,3, 4,5} . Ta thấy rằng các tam giác thứ i, i + 1, i + 2 với i xét theo
modulo 5 không thể cùng được xuất hiện trong B nên có thể giả sử B = {1, 2, 4} và khi đó, ta có
tập hợp A = {3,5, 6, 7,8,9,10} .
Suy ra AR
= RQ
= QD, BN
= NP
= PD dẫn đến PQ  NR  AB và ER = BR nên ER > MB , mâu
thuẫn.
(2) Trường hợp hai tam giác thuộc {1, 2,3, 4,5} và một tam giác thuộc {6, 7,8,9,10} , không mất
tính tổng qt, giả sử 6 ∈ B thì B = {3,5, 6} , suy ra QR là đường trung bình của tam giác EMP
và PQ  MP , tương tự NP  MQ nên tứ giác MPDQ là hình, suy ra MA = MC. Tương tự, ta
cũng có MB = ME hay ABCE là hình bình hành, dẫn đến
=
DQ

DQ PQ 1
. Tiếp tục suy ra
= =
DA AB 3

1
=
DA, DQ AR nên DQ

= QR
= RA , mâu thuẫn.
3

Do đó, trường hợp 7 tam giác có diện tích đơi một bằng nhau không thể xảy ra và ta chỉ cần chỉ
ra một trường hợp có 6 tam giác có diện tích đôi một bằng nhau thỏa mãn.

A
4

1
D

2
3

E

M P
N

Q
R

B
5
6

C


Xây dựng ngũ giác như hình vẽ: tam giác ACE cân tại A và AM
= MN
= NE , AP
= PQ
= QC .
Dễ thấy rằng khi đó các ngũ giác 1, 2,3, 4,5, 6 có diện tích đơi một bằng nhau.
Vậy giá trị lớn nhất cần tìm là 6.
Đây là bài tốn khá thú vị. Chúng ta có thể thay yếu tố diện tích bởi yếu tố chu vi để có một bài
tốn dạng tương tự khác (nhưng khó hơn nhiều).

22


Bài 17 (Đề kiểm tra trường hè 2014) Có 2014 đường thẳng l1 , l2 ,..., l2014 nằm trên mặt phẳng
sao cho khơng có 2 đường thẳng nào song song với nhau, khơng có 3 đường thẳng nào đồng quy.
Chứng minh rằng tồn tại một đường gấp khúc A0 A1... A2014 gồm 2014 đoạn thẳng nhỏ, sao cho
đường gấp khúc này khơng tự cắt chính nó, và ứng với mỗi k ∈ , k ≤ 2014 thì tồn tại i sao cho
đoạn Ai Ai +1 nằm trọn trên lk .
Giải.
Dưới đây là một cách xây dựng thỏa mãn:
Gọi S là tập hợp các giao điểm của 2014 đường thẳng ban đầu:
(i) Điểm A0 được chọn là điểm nằm trên một đường thẳng bất kì trong các đường thẳng ban đầu
nhưng không thuộc S.
(ii) Điểm A1 được chọn sao cho:
(a) A1 ∈ S .
(b) A0 A1 là một đường thẳng trong 2014 đường thẳng ban đầu.
(c) A0 A1 có chiều dài ngắn nhất trong các trường hợp thỏa (a) và (b).
(iii) Giả sử nếu đã xây dựng được các điểm A0 , A1 ,..., An với 1 ≤ n ≤ 2012 thì điểm An +1 được
chọn sao cho:
(a) An +1 ∈ S .

(b) An An +1 là một đường thẳng trong 2014 đường thẳng ban đầu.
(c) An +1 ∉ Ai Ai +1 , ∀0 ≤ i ≤ n − 1 .
(d) An An +1 có chiều dài ngắn nhất trong các trường hợp thỏa (a), (b), (c).
Trước tiên, ta thấy rằng cách dựng như trên là có thể thực hiện.
Kế đến, sử dụng điều (c) của ý (iii) để chứng minh rằng đường gấp khúc A0 A1... A2013 đã đi qua
2013 đường thẳng phân biệt (trong 2014 đường thẳng ban đầu). Việc còn lại là sử dụng điều kiện
“ngắn nhất” để chứng minh rằng A0 A1... A2013 không tự cắt.
Cuối cùng, chỉ cần xây dựng điểm A2014 là bài toán kết thúc.

23


Bài 18. (China TST 2012) Trong một hình vng được tạo bởi 2012 × 2012 ơ vng có chứa
những con bọ, trong một ơ vng có nhiều nhất 1 con bọ. Vào một thời điểm nào đó, những
con bọ này bay lên rồi lại đậu xuống lại vào các ô vng, mỗi ơ vng cũng có nhiều nhất
một con bọ. Đối với mỗi con bọ, có thể xem đoạn thẳng có hướng nối tâm của ơ vng lúc đầu
và tâm của lúc sau mà nó đậu lên tạo thành một vector. Với một số lượng bọ ban đầu, xét tất
cả những trường hợp có thể xảy ra với các vị trí đầu tiên và cuối cùng của những con bọ, hãy
tìm độ dài lớn nhất của tổng các vector.
Giải.
Xét mặt phẳng chứa hình vng đang xét là mặt phẳng tọa độ Oxy với O là tâm của hình vng
lớn. Gọi tập hợp các tâm của tất cả các hình vng nhỏ là S, tập hợp những điểm những con bọ
dừng lại lúc đầu là M 1 ⊆ S , tập hợp những điểm những con bọ rơi xuống lúc sau là M 2 ⊆ S .
Bỏ qua trường hợp một con bọ ở cùng một nơi lúc đầu và lúc sau. Xét phép đặt tương ứng từ M 1
đến M 2 là f .
Để đơn giản, với một điểm x ∈ S , ta kí hiệu x là vector có điểm đầu là O, điểm cuối là x. Ta
có tổng các vector được tính bằng V=

∑ ( f (v) − v=) ∑ u − ∑ v .


v∈M1

u∈M 2

u∈M1

M 2 ; đồng thời, do ta cần tìm giá trị lớn nhất của độ dài của
Dễ thấy rằng M 1 , M 2 ⊆ S , M 1 =
vector V nêu trên nên chỉ cần xét trường hợp M 1 ∩ M 2 =
∅ (bởi vì nếu M 1 , M 2 có giao khác
rỗng thì phần giao của chúng sẽ cho các vector có độ dài bằng 0).
Do cách chọn M 1 , M 2 hữu hạn nên sẽ tồn tại một cặp ( M 1 , M 2 ) nào đó mà V đạt giá trị lớn
nhất và hiển nhiên giá trị V đó khác 0. Qua O kẻ đường thẳng l vng góc với V.
Ta sẽ chứng minh các bổ đề sau:
Bổ đề 1.
Nếu V đạt giá trị cực đại thì đường thẳng l khơng đi qua bất kì điểm nào của S và M 1 , M 2 nằm
ở hai phía khác nhau so với l.
Chứng minh.
Đầu tiên ta thấy rằng M 1 ∪ M 2 =
S vì nếu khơng thì do S = 20122 là số chẵn nên có ít nhất hai
điểm a, b khơng có trong M 1 ∪ M 2 , có thể giả sử góc tạo bởi a − b và V khơng q 900 thì

24


V + (a − b) > V (nếu góc đó lớn hơn 900 thì xét b − a ). Suy ra nếu thêm a vào M 2 , thêm b
vào M 1 (hoặc ngược lại) thì V tăng lên, mâu thuẫn do V đạt giá trị cực đại.
Hơn nữa, M 2 = − M 1 vì nếu khơng thì trong M 1 , M 2 đều có chứa ít nhất một cặp điểm đối xứng,
cụ thể là a, b ∈ S sao cho a, −a ∈ M 1 ; b, −b ∈ M 2 .
Có thể giả sử a − b và V hợp thành góc khơng q 900 thì




u∈( M 2 \{b}) ∪{a}

u−



v = V + 2(a − b) > V .

v∈( M1 \{a}) ∪{b}

Điều này cũng mâu thuẫn với tính lớn nhất của V hay M 1 , M 2 là tập hợp các điểm đối xứng với
nhau.
Ta cũng có l khơng đi qua bất kì điểm nào thuộc S vì nếu khơng thì do l đi qua O nên nếu l đi
qua một điểm a ∈ M 1 thì nó cũng đi qua −a ∈ M 2 . Khi đó, ta đổi chỗ, thay a vào M 2 , thay −a
vào M 1 thì tổng vector là V + 4a , chú ý rằng góc hợp bởi a và V là 900 nên V + 4a > V , mâu
thuẫn.
Cuối cùng, ta cũng phải có M 2 là tập hợp các điểm nằm về một phía so với l , giả sử đây là phía
có chứa điểm cuối của vector V và M 1 nằm về phía ngược lại vì nếu khơng thì về phía có điểm
cuối của vector V , tồn tại hai điểm a, b lần lượt thuộc M 1 , M 2 thì góc hợp bởi a − b và V nhỏ
hơn 900 ; tương tự, ta lại thay a vào M 2 , b vào M 1 thì sẽ có
V + 2(b − a ) > V , mâu thuẫn với tính lớn nhất của V .

l
V

O


25


×